Sei sulla pagina 1di 46

HISTORY

1. LOW BACK PAIN

CASE 1.
LOW BACK PAIN
This patient has been referred as an emergency to the surgical assessment unit. The GP has said that the
patient has back pain which has worsened today and they are unable to control the pain in primary care. You
are to take an appropriate history in 6 minutes (you may make notes). If you complete your history within the
6 minutes you should indicate to the examiners that you are ready. In the remaining 3 minutes the examiners
will ask you to present a summary of the history. They may also ask you to discuss any particular physical
signs you would look for on examination, the likely differential diagnosis, appropriate investigations and a
management plan.

HISTORY QUESTIONS TO ASK


Introductions:

 ‘Hello, my name is Dr Parchment Smith. Can I please check your details?’


 ‘Would you mind if I asked you some questions about your back pain?’

Ask about demographics:

 ‘May I ask how old you are?’


 ‘What do/did you do for a living/hobby?’

Presenting complaint (open question):

 ‘Can you tell me about what has caused you to come and see me today?’

History of presenting complaint (the patient may volunteer the following information but remember a
systematic approach to taking a pain history is SIROD CASP):

 Site: ‘Where do you feel the pain?’ (localised or diffuse).


 Intensity: ‘Can you rate the pain on a scale from 1 to 10, with 1 being no pain and 10 the worst pain
ever?’ This is subjective and it is more important to know how it affects their life, eg ‘Does the pain
limit your daily activities?’ If yes: ‘How does it restrict you?’, eg ‘How far can you walk?/do you require
a walking aid?’
 Radiation: ‘Does the pain move anywhere else?’ If it moves into the legs: ‘How far down the leg?’
 Onset: ‘Did the pain come on suddenly or gradually?’
 Duration: ‘How long does the pain last?’ (constant or related to position/use).
 Character: ‘What is the pain like?’ (dull, aching, sharp or knife-like).
 Alleviating and exacerbating factors: ‘Does anything make the pain better, or worse?’ In particular
ask: ‘Is the pain better or worse with exercise?’ ‘Is the pain worse at any particular time of day?’
 Symptoms associated with the pain: ‘Do you notice anything else when you get the pain?’ (eg
weakness, wasting, fibrillation, gait problems, foot drop, ankle giving way).
 Previous episodes: ‘Have you had a pain like this before?’ If yes: ‘What did you do then?’

You must ask about associated symptoms that are other red flag symptoms:

 ‘Have you had any problems with passing urine lately?’


 ‘Have you had any problems with your bowels lately?’
 ‘Have you developed any weakness?’ If yes: ‘Where do you feel this?’ ‘Is it getting any better or worse?’
 ‘Do have any loss of sensation?’ If yes: ‘Where do you feel this?’ ‘Is it getting better or worse?’
 ‘Do you have a fever associated with the pain?’
 ‘Have you lost any weight recently?’

Past medical and surgical history:

 In particular, ask what investigations/treatment have they had for the pain.
 Have they other diseases which are risk factors for back disease (eg red flag medical history would
include carcinoma, immunosuppression, eg HIV or diabetes, osteoporosis)?
 Have they a history of injury?
 ‘Have you had any tests to investigate the pain?’ ‘Have you had any treatment for it?’ ‘Do you have any
other medical problems?’ ‘Have you injured your spine?’

Medication and allergies:

 ‘Do you take any regular medication?’


 ‘What medication, if any, do you take for your back pain?’
 ‘Do you have any allergies?’

Family history:

 ‘Do any diseases run in the family?’

Social history (smoking and drinking):

 ‘Are you a current or ex-smoker?’


 If yes, ask about smoking habits: ‘What do you smoke?’ ‘How many do you smoke a day?’ ‘How long
have you smoked for?’
 ‘Do you, or have you ever, drank alcohol?’
 If yes: ‘What type of alcohol do you drink?’ (eg spirits, wine, beer). ‘How much do you drink per week?’
(convert this to units per week). ‘How long have you been drinking for?’

DETAILS

 The patient is a 40-year-old man who has presented to the Emergency Department. He suffers with
chronic lower back pain which has been ongoing for the past 10 years. He works as a cleaner and 2
days ago, after bending down, he began experiencing shooting pain down his right leg.
 Over the past 2 days he also had a shooting pain down his left leg. When asked, he also mentions that
he has had a 12-hour history of not being able to pass urine and that there is numbness around his
seat (bottom). He has no bowel issues. Also when asked he discloses that the pain is persistent down
his legs.
 On direct questioning, he said that he has no morning erection/erectile dysfunction (this is a key
question for the candidate to ask as an opportunity to demonstrate to a lay examiner that they
can ask intimate questions in a sensitive way with signposting, etc). He also discloses when asked
that he has in the past experienced a similar shooting pain in the right leg previously which resolved.
 The past medical history is chronic lower back pain and hypertension.
 There is no family history.
 Social history: smoker for 20 years.

Q. What would be the most concerning symptoms in a patient with back pain.
The most concerning symptoms would be neurological symptoms:
Urinary retention,
Saddle anaesthesia and
Anaesthesia of the lower dermatomes.

Q.Some symptoms patients may be unwilling to divulge.


On direct questioning there was no morning erection/erectile dysfunction. This is a key question opportunity
to demonstrate to the examiner that you can ask intimate questions in a sensitive way with signposting, etc
Q. Please summarise the history.
This is a 40-year-old man with an acute exacerbation of chronic pain. This came on suddenly at work and has
resulted in symptoms of cauda equina syndrome, including saddle anaesthesia, urinary retention and erectile
dysfunction. He has had previous sciatic-like pain but without neurological symptoms.

Q. Please give a differential diagnosis for this patient.


 The most concerning diagnosis is Cauda equina syndrome.
 Sciatica with Urinary retention due to pain is another differential at this stage.

Q.Please explain how the diagnosis would be confirmed.

 A full examination is required, particularly looking for perianal sensory loss and anal tone.
 I would carefully check for a reduction in power and decreased reflexes in a Dermatomal/myotomal
pattern.
 I would look for sciatic nerve irritation.
 Routine bloods would be required.
 The patient needs to be admitted for MRI if cauda equina is suspected.

Q. Please specify what management you would recommend.


If worried about cauda equina syndrome, my next steps would be:

 To ask for a post-void bladder scan.


 Arrange an urgent lumbar MRI, which needs to be done today.
 Immediately discuss the case with the spinal surgeons for their advice and/or ongoing management.
 The patient might need an emergency lumbar decompression.

If the patient has sciatica, management would involve:

 Refer the patient to a spinal surgeon (neurosurgeon or orthopaedic surgeon).


 Administer adequate analgesia.
 Perform an MRI lumbar spine within the next 2 months.
 Perform nerve conduction studies.
 It is important to refer the patient on or review them again once they have had these investigations. It
might be appropriate to refer to a pain specialist; however, you would probably await the MRI results
first.

Key knowledge:

 Different nerve roots are affected by central disc versus lateral disc protrusion.
 Damage to the tough outer layer of the disc (annulus fibrosis) is usually due to degeneration, which
allows the soft central nucleus pulposus to herniate. Disc prolapse is usually either central or lateral.
 A lateral disc prolapse is most likely to affect the exiting nerve root of the level above, while a central
disc prolapse is most likely to affect the exiting nerve root of the level below.
 Nerve root compression can lead to pain, numbness, muscle wasting, weakness and paraesthesiae.
 In a degenerate spine, there may be multilevel disease and so a good knowledge and understanding of
dermatomes and myotomes is essential for correlating clinical and radiological findings.
CASE.2

DYSPHAGIA

Mr D, a 75-year-old retired bus driver, has come to see you in clinic as he is having problems with swallowing.
Please take a history and list your differential diagnoses. You are to take an appropriate history in 6 minutes
(you may make notes). If you complete your history within the 6 minutes you should indicate to the examiners
that you are ready. In the remaining 3 minutes the examiners will ask you to present a summary of the history.
They may also ask you to discuss any particular physical signs you would look for on examination, the likely
differential diagnosis, appropriate investigations and a management plan.

HISTORY BY PATIENT
What you need to know:
Dysphagia refers to difficulty (not pain) when swallowing;
Pain on swallowing is called Odynophagia.
Introductions:

 ‘Hello, my name is Dr Parchment Smith. Can I check your details please? Would you mind if I asked you
some questions?’

Ask about demographics:

 ‘May I ask how old you are?’


 ‘What do/did you do for a living?’

Presenting complaint (open question):

 ‘What has caused you to come and see me today?’

History of presenting complaint (the patient may volunteer the following information but questions you need
to cover include):

 ‘Have you been getting difficulty with swallowing?’(Given the subject, hopefully the answer will be
yes!)
 ‘How long has this been going on for?’
 ‘Is it constant/intermittent?’
 ‘Is it painful to swallow?’ (ie odynophagia rather than dysphagia).
 ‘Is the pain constant or intermittent?’
 ‘Does any particular type of food or drink cause the pain?’
 ‘Has it been getting progressively worse?’
 ‘Can you swallow solids, liquids, your saliva?’
 ‘Can you drink fluid as fast as you used to?’
 ‘Is it difficult to make a swallowing movement?’
 ‘Does food seem to get stuck anywhere in particular?’
 ‘Do you ever bring food back up?’ (reflux of food is effortless, not vomiting).
 ‘Do you get a sensation of a lump in your throat?’

Associated symptoms to enquire about include:

 ‘Have you ever noticed a lump on your neck?’ If yes, possibilities include metastatic lymphadenopathy
from oesophageal or laryngeal/pharyngeal carcinoma or a pharyngeal pouch; if this is a possibility
enquire: ‘Does your neck bulge or gurgle on swallowing?’
 ‘Have you noticed a change in your voice?’
 ‘Have you noticed any problems with your breathing?’ (As mentioned earlier you are very unlikely to
see a patient with stridor in the exam but you need to be seen to be asking about their airway.)
 ‘Have you noticed any weight loss?’ If yes, try to quantify how much and over what time period. This
could obviously be caused by the dysphagia itself as well as by the underlying cause for the dysphagia.
 ‘Have you developed a cough?’
 ‘Have you got any pain elsewhere?’ (epigastric pain/heart burn – gastro-oesophageal reflux disease
(GORD); otalgia – referred pain from pharyngeal carcinoma).

Past medical and surgical history:

 In particular, what investigations have they had for the dysphagia? Have they had any
operations/radiotherapy/chemotherapy? Are they requiring nutritional supplementation; if yes, what
kind?
 ‘Have you had any investigations for your difficulty in swallowing?’
 ‘Have you had any treatment for your difficulty in swallowing?’

Medication and allergies:

 Do they take any medication for reflux/GORD?’ ‘Are you on any medication?’ ‘Do you have any
allergies?’

Social history (smoking and drinking):

 It is important to ask about smoking as this is a risk factor for GORD.


 ‘Are you a current or ex-smoker?’ If yes, ask about smoking habits: ‘What do you smoke?’ ‘How many
do you smoke a day?’ ‘How long have you smoked for?’
 ‘Do you, or have you ever, drank alcohol?’ If yes: ‘What type of alcohol do you drink?’ (eg spirits, wine,
beer).
 ‘How much do you drink per week?’(convert this to units per week). ‘How long have you been drinking
for?’

Dysphagia

Mechanical causes of dysphagia:

 Mechanical causes due to a structure blocking the oesophagus include differentials such as
oesophageal/pharyngeal malignancy, cricoid web, peptic stricture, GORD, extrinsic pressure from
lymphadenopathy or bronchial carcinoma.
 The cause in this patient is oesophageal carcinoma.
 The geographical distribution of oesophageal carcinoma is that squamous cell carcinoma is endemic in
Africa/Asia while
 Adenocarcinoma is commoner in westernised countries.
 There is a metaplastic change if the lower oesophagus changes from squamous to columnar-type
epithelium in response to GORD, which results in Barrett’s oesophagus. Metaplasia increases the risk
of developing adenocarcinoma approximately 25 times.

Treatment options:

 If it is a curable disease treat with surgery, eg oesophagostomy/oesophagogastrostomy, with or


without chemotherapy.
 If it is incurable, options include nutritional support, stenting, endoscopic laser therapy and
chemoradiotherapy.

Motility disorders causing dysphagia:

 Disorders include achalasia, diffuse oesophageal spasm and CREST syndrome (calcinosis, Raynaud’s
phenomenon, oesophageal dysmotility, sclerodactyly and telangiectasia).

Dysphagia can be classified into oropharyngeal, oesophageal, systemic and functional. When considering
causes of oesophageal dysphagia, the candidate should structure differentials into: causes outside the lumen,
within the wall of the lumen, and within the lumen itself.
A barium swallow would demonstrate a tapering at the lower oesophageal sphincter, with dilatation of
the oesophagus above this. This is known as the ‘bird’s beak sign’.
Patient (actor) briefing

 Behaviour: tired.
 History of presenting complaint: he first noticed a swallowing issue several months ago when eating
bread. It seems to be getting worse. It is worst with thick foods such as meat and bread, but liquids
such as soup and ice-cream are fine. Food seems to get stuck in the same place each time. There is no
pain but it is getting in the way of normal eating as when it gets stuck the food must be brought back
up. It is more like regurgitating the food, not vomiting. He has lost 2 stones (12.7 kg) since he began
having the swallowing difficulty. He has no cough and no pain anywhere else. There is no gurgling or
bubbling when eating..
 Past medical history: nil.
 Past drug history: nil.
 No known drug allergies.
 Social history: lives with his wife.
 Smoked 20 cigarettes a day for 40 years. Drinks around 20 units of alcohol per week.

Q.1.Any red flags for malignancy.


Red flags are slow-onset dysphagia solids then liquids, weight loss, smoking history and alcohol history.

Q.1.Please summarise the history.


This is a 75-year-old man with a 40-year history of smoking 20 cigarettes a day, presenting with progressive
dysphagia over several months, which is worse with solids but liquids are OK, is now regurgitating food and
has a 2-stone (12.7 kg) weight loss. There are no obvious neurological symptoms.
Q.2.Please give a differential diagnosis and explain how you would justify the answer.
Mr D’s symptoms are fitting with a mechanical obstruction. Given that the patient has had a certain amount of
‘unintentional weight loss’, it is of paramount importance to exclude a malignant cause for his symptoms. He
presents with a gradual onset of dysphagia for solids over liquids, has suffered significant weight loss and has
a significant smoking history making oesophageal carcinoma the top differential.
Mechanical causes due to a structure blocking the oesophagus include differentials such as:

 Oesophageal/Pharyngeal malignancy
 Cricoid web
 Peptic stricture
 Gastro-oesophageal reflux disease (GORD)
 Extrinsic pressure from lymphadenopathy or bronchial carcinoma
 Pharyngeal pouch.

Functional or mobility causes include differentials such as:

 Motor neurone disease


 Post-cerebral vascular accident
 Globus
 Diffuse oesophageal spasm
 Scleroderma.

Q.3.Please specify what management you would recommend.


If oesophageal cancer is confirmed, management involves
Surgical resection after discussion in a multidisciplinary team (MDT).
The early involvement of oncology specialists allows planning of neoadjuvant and adjuvant
chemoradiotherapy.
Palliative care in cases not amenable to surgery (including stenting).
The two main surgical options are a
 Transhiatal oesophagectomy (THE) via an abdominal and cervical junction incision with blunt
mediastinal dissection through the oesophageal hiatus, or a
 Transthoracic oesophagectomy (TTE) via an abdominal and a right thoracic incision.
Q.4.INVESTIGATIONS:

 UPPER GI ENDOSCOPY
 IF MOTILITY DISORDER: CONTRAST STUDY BY BARIUIM SWALLOW
 STAGING: CT

CASE .3.HAEMATURIA

You are the CT2 covering the urology clinic today. You have been asked to take a history from a 64-year-old
man, a retired shop worker, who has been referred by his GP with haematuria. You are to take an appropriate
history in 6 minutes (you may make notes). If you complete your history within the 6 minutes you should
indicate to the examiners that you are ready. In the remaining 3 minutes the examiners will ask you to present
a summary of the history. They may also ask you to discuss any particular physical signs you would look for
on examination, the likely differential diagnosis, appropriate investigations and a management plan.

 History of presenting complaint: 64-year-old-man. First noticed bloodstained urine 3 months ago,
throughout urination, but he only visited the GP when back pain developed. The back pain is mainly
left. The blood seems to be early in the stream and the back pain is a dull ache in the flank. It is not
made worse by movement, there is no radiation down legs and no neurological deficit.
 Urinary symptoms: he is going to the bathroom during the night, making more trips to the toilet, and
sometimes he must rush to get to the bathroom but he has not been incontinent. It takes some time for
him to begin urinating and the stream of urine is very weak. There is sometimes dribbling after
passing urine.
 He is feeling tired and has lost weight unintentionally.
 Past medical history: nil.
 Past drug history: nil.
 He has no known drug allergies.
 Social history: lives with his wife, does not smoke and does not drink any alcohol, and has had no
recent foreign travel.
 Industrial risk factors: nil.
 Family history: nil. Systems review: nil other.
Ask about lower urinary tract symptoms:

 These can be storage symptoms, eg frequency, nocturia, dysuria, urgency.


 ‘Are you passing urine more frequently than usual?’
 ‘Do you get up at night to pass urine?’ If yes: ‘How many times?’
 ‘Does it sting or burn when you pass urine?’
 ‘Do you have to rush to get to the toilet to pass urine?
 ‘Have you ever been incontinent of urine or had voiding symptoms, eg hesitancy, straining, poor
stream, terminal dribbling?’
 ‘Does it take a while for you to start urinating?’
 ‘Do you have to strain?’
 ‘Is your stream less powerful than it was?’
 ‘Do you find you dribble at the end of urination?’
 ‘Have you ever had retention of urine – when you have been unable to pass urine and required a
catheter?’

Then ask about other associated symptoms:

 ‘Have you ever passed air or faecal matter in your urine?’


 ‘Have you been getting any pain – where do you feel this?’ (SIROD CASP).
 ‘Have you lost any weight?’ ‘Have you had a fever?’

Past medical and surgical history:

 In particular, what investigations have they had for the haematuria? Have they had any procedures or
operations?
 ‘Have you had any tests to investigate the blood in the urine?’
 ‘Have you had any treatment for it?’

Any risk factors for haematuria.

 Smoking
 Industrial: dye workers, rubber factory workers
 Schistosomiasis areas (Egypt, East Africa).

The red flags for malignancy.

 Weight loss
 Back pain
 Tiredness (Anemia)

A 64-year-old man with a 3-month history of frank haematuria. The haematuria has been worsening and
begins on urination. He has nocturia, frequency and urgency. Left flank/back pain has developed over this
time, as has malaise and fatigue. He has lost weight over this period unintentionally. He does not smoke or
drink and has no industrial risk factors for urinary malignancy.
Q. Please give a differential diagnosis and explain how you would justify the answer.

 Bladder carcinoma (in the top 2 differential diagnoses)


 Kidney (transitional cell carcinoma (TCC)/ renal cell carcinoma (RCC)) (in the top 2 differential
diagnoses)
 UTI
 Renal stones
 Bladder stones
 Prostate carcinoma.
The unintentional weight loss and tiredness are likely a result of malignancy and the back pain could be a
result of metastatic disease.
Q. Please explain how the diagnosis would be confirmed.
Urgent investigations (2-week rule) are required for patients with macroscopic haematuria or those >50 years
with microscopic haematuria. At the haematuria clinic:

 DRE prostate exam


 Urine cytology
 Urine culture
 Ultrasound scan or CT IVU to image upper urinary tract
 Flexible cystoscopy.

Q. Please specify what management you would recommend.


An urgent multidisciplinary team (MDT) discussion would be required following the diagnosis of malignancy.
Depending on the stage of malignancy a surgical or conservative approach would be considered. This would
include:

 Transurethral resection of bladder tumor (TURBT)


 Radical cystectomy
 Nephrectomy
 Chemotherapy as an adjunct or as part of a conservative program.
CASE.4

HEADACHE-1

You are the SHO on call overnight covering all surgical specialties. A&E refer Mrs H, a 46-year-old bartender,
presenting with a severe headache. You are to take an appropriate history in 6 minutes (you may make notes).
If you complete your history within the 6 minutes you should indicate to the examiners that you are ready. In
the remaining 3 minutes the examiners will ask you to present a summary of the history. They may also ask
you to discuss any particular physical signs you would look for on examination, the likely differential
diagnosis, appropriate investigations and a management plan.

Patient (actor) briefing


Select 'Next' to see.

 History of presenting complaint: it came on suddenly at the back of her head while at work (she is a
bartender). This is the worst headache ever. Headache has persisted and now she is finding the light
slightly uncomfortable. She has vomited twice. There has been no loss of consciousness and no
seizures. No previous head trauma. Otherwise well, no other symptoms.
 Behaviour: not confused but the pain is making it hard to concentrate.
 Past medical history: hypertension.
 Past drug history: amlodipine, no known drug allergies.
 Social history: she lives with her husband and two children. She smokes 25 cigarettes a day and
consumes 20 units of alcohol per week.
 Family history: mother died at the age of 45 from a brain problem (you are not sure exactly).
 Ideas, concerns and expectations: she is worried she is going to die because of what happened to her
mother.
 Systems review: nil other.

Consider...
The danger signs for headaches.

 Thunderclap headache
 Vomiting
 Seizures
 Loss of consciousness
 Altered vision
 Photophobia
 Worse on increased intracranial pressure (sneezing, bending down).

Q. Please summarise the history.


This is a 46-year-old bartender with occipital thunderclap headache. She has vomited twice, is photophobic,
but has not lost consciousness or seized. Her past medical history is hypertension and she is a smoker. She has
a family history suspicious for subarachnoid haemorrhage.
Q. Please give a differential diagnosis and explain how you would justify the answer.
The main differential is a spontaneous Subarachnoid haemorrhage (SAH), but I would also consider other
causes of an acute severe headache including:

 Meningitis.
 Encephalitis.
 Cerebral sinus thrombosis
 Arteriovenous malformation (avm) rupture
 Intracerebral haemorrhage.
 Migraine.

Q. Please specify what management you would recommend.

 Initial resuscitation (ABC approach) if the patient is compromised.


 Ensure the patient has adequate analgesia using the analgesic ladder.
 Obtain baseline blood tests including surgical blood tests.
 Regular neurological observations.
 Organize imaging: CT head and then CT angiogram if abnormalities on CT head.
 Ensure the patient is NBM in case she is required to have an operation.
 Inform neurosurgical team urgently of the patient and scan findings.
 Follow neurosurgical advice on immediate management of patient (may include laxatives and
nimodipine).
 Inform ITU/anesthetic teams.

Aneurysm:

 This is predominantly treated by endovascular coil embolization or vascular stenting.


 Surgical management includes clipping the aneurysm.

Q. What are the red flag symptoms that the patient presented with and what are two other risk factors not
mentioned?
Red flags:

 Worst ever headache


 Meningism
 Smoker
 Family history
 Hypertension.

Risk factors:

 Connective tissue disorders (eg Ehlers–Danlos syndrome)


 Autosomal dominant polycystic kidney disease
 Drug use (particularly cocaine).

Q. What are the causes of a SAH?


The main causes of SAH are:

 Traumatic (post head injury)


 Spontaneous:

 Aneurysmal
 Peri-mesenecaphlic haemorrhage
 Arteriovenous malformation
 Idiopathic
 Bleeding disorders
 Dissection of the vertebral artery.

Risk factors for spontaneous cases included ;


 High blood pressure
 Smoking
 Family history
 Alcoholism
 Cocaine
 Autosomal dominant polycystic kidney disease
 marfan

Specific types of headache:

 Tension headache: the commonest form of headache experienced. It is a diffuse, dull, aching with a
‘tight band’, and patient may have concomitant depression or stressors.
 Migraine: can exist with or without aura, characterised by a unilateral throbbing headache. It is more
common in women, often familial, there are various subtypes, and it is managed by neurologists.
 Cluster headache: severe unilateral pain around one eye. It is more common in men, with onset in
middle age, and can be associated with conjunctival injection, lacrimation, rhinorrhoea and,
occasionally, a transient Horner’s.
 Giant cell (temporal) arteritis: idiopathic, autoimmune disease that affects the elderly, and most
commonly affects the superficial temporal artery.
 Post-traumatic headache: headache similar in character to migraine or tension-type headache
following head injury. It can be associated with poor concentration, irritability and emotional lability.
 Low-pressure headache: post lumbar puncture, occult CSF leak following epidural anaesthesia,
overdrainage of CSF in shunted patients. It is worse when erect and eased by lying flat.
 Idiopathic intracranial hypertension: most common in overweight young women. It is
characterised by diffuse aching pain, associated with deterioration in visual acuity and papilloedema
on fundoscopy.
CASE 5
HEADACHE HISTORY 2
You are the on-call SHO for neurosurgery. Mr S, 80 years old and retired, presents to the Emergency
Department with headaches. You are to take an appropriate history in 6 minutes (you may make notes). If you
complete your history within the 6 minutes you should indicate to the examiners that you are ready. In the
remaining 3 minutes the examiners will ask you to present a summary of the history. They may also ask you to
discuss any particular physical signs you would look for on examination, the likely differential diagnosis,
appropriate investigations and a management plan

Patient (actor) briefing.

 History of presenting complaint: headaches are worse when bending and straining. About 2–3 months
ago he hit his head on a shelf when cleaning. He didn’t think much of it at the time but symptoms have
been worsening since the incident. No other weakness or symptoms. Daughter has noticed that he has
also become unsteady on his feet.
 Past medical history: Atrial fibrillation and Ischaemic heart disease.
 Drug history: warfarin and simvastatin, no known drug allergies.
 Social history: does not smoke or drink alcohol.
 Lives alone: independent in terms of activities of daily living.
 Family history: nil.
 Ideas, concerns and expectations: worried he is going to die.
 Systems review: nil.

HISTORY:
What you need to know:
Headaches are very common; an extremely small percentage of patients presenting with this symptom will
have a serious underlying surgical condition. The key to a good history is identifying ‘red flag’ signs that may
warrant further investigation, with the ultimate aim of ruling out (or in) a space-occupying lesion. It is also
worth noting that many brain tumours will not present with headache as the primary complaint.
Introductions:

 ‘Hello, my name is Ryan Mathew. Please can I check your details?'


 'Would you mind if I asked you some questions about your symptoms?’

Ask about demographics:

 ‘May I ask how old you are?’


 ‘Are you right- or left-handed?’
 ‘What do/did you do for a living?’

Presenting complaint (open question):

 ‘What has caused you to come and see me today?’


 ‘How long have you had headaches for?’ (days or weeks).
 ‘Is there anything that might have brought it on?’
 ‘Can you point on your head where the pain is worst?’ ‘Does the pain go anywhere?’
 ‘Have you ever had a headache that was sudden and very severe?’
 ‘Are the headaches worse at any time of the day?’ ‘Do they get worse with sneezing, straining or coughing?’
 ‘Do you know when the headaches are going to come on?’
 ‘Do you notice anything else when the headaches come on, such as changes to your vision?’
 ‘When did you last visit an optician?’ ‘Do you notice if your headaches come on when straining your eyes,
such as watching TV or using the computer for a long time?’
 ‘Have you ever had a seizure or blacked out?’ ‘Have you noticed any weakness or numbness in your arms
or legs?’

Past medical and surgical history:

 ‘Have you ever had headaches in the past?’


 ‘Have you ever had any brain operations?’ ‘Have you ever had chemo/radiotherapy?’

Medication and allergies:

 ‘Do you take any regular medication?’ ‘Do you have any allergies?’

Family history:

 ‘Has anyone in your family ever suffered from headaches?'


 'Has anyone in your family ever had an operation on their brain?’

Social history (smoking and drinking):

 ‘Do you smoke?’ ‘Do you take any recreational drugs?’ (if relevant).

Headache combined assessment:

 insidious or sudden onset?


 length of history? diurnal variation?
 associated symptoms, eg seizures, visual symptoms, limb weakness/sensory disturbance, level of
consciousness (LOC), aura.

Examine:

 full cranial nerve examination


 fundoscopy
 peripheral neurological examination including cerebellar function

 The symptoms of increased ICP.


Symptoms are:

 Worse on bending over


 Often worse in the morning
 Deterioration in vision
 High blood pressure.

Q. Please summarise the history.


This is an 80-year-old retiree who has a 3-month history of headaches following head trauma. The headaches
are worse on straining and bending over. He has associated symptoms of unsteadiness on his feet.
Significantly he is on warfarin for atrial fibrillation (AF).
Q. Please give a differential diagnosis and explain how you would justify the answer.
The features of the headache seem to be related to Raised intracranial pressure (ICP); therefore the headache
could be due to a space-occupying lesion:

 Meningioma
 Glioma
 Abscess
 With the history of previous trauma and slow decline and his anticoagulation medication, especially in
this patient’s age group, a CHRONIC SUBDURAL HAEMATOMA is the most likely diagnosis.

Q. Please specify what management you would recommend.

 Full history and examination (neurological/Glasgow Coma Score (GCS)).


 I would perform initial resuscitation (ABC approach) if the patient is compromised.
 I would ensure the patient has adequate analgesia using the analgesic ladder.
 I would obtain baseline blood tests including surgical blood tests (especially clotting).
 I would obtain a plain CT head.
 The patient should be kept NBM in case they need emergency surgery.

Q. The patient is found to have a chronic subdural haematoma on the right side, with 'midline shift' (MLS) and
their INR was 4. What is MLS? What would you do next?

 MLS refers to deviation of the intrahemispheric fissure away from the midline.
 The degree of MLS is significant as it gives an indication of compression on the brain/brainstem.
 I would reverse the patient’s anticoagulation: given the need for urgent surgery: beriplex or 10 mg of
IV vitamin K would be most appropriate. Guidelines for reversing anticoagulation can be found in local
policy and the BNF.
 I would discuss the case with the local neurosurgical centre.

Subdural
Hematoma type Epidural

Between the skull and the outer Between dura mater and arachnoid
Location
endosteal layer of the dura mater mater.
Temperoparietal locus (most likely) -
Middle meningeal artery
Frontal locus - anterior ethmoidal artery
Involved vessel Bridging veins
Occipital locus - transverse or sigmoid
sinuses
Vertex locus - superior sagittal sinus
Symptoms (depend on Lucid interval followed Gradually
severity)[11] by unconsciousness increasing headache and confusion
CT appearance Biconvex lens Crescent-shaped
CASE 6

HEADACHE HISTORY 3

You are the neurosurgery SHO in clinic. Mrs B is a 54-year-old shop owner with intermittent headaches. You
are to take an appropriate history in 6 minutes (you may make notes). If you complete your history within the
6 minutes you should indicate to the examiners that you are ready. In the remaining 3 minutes the examiners
will ask you to present a summary of the history. They may also ask you to discuss any particular physical
signs you would look for on examination, the likely differential diagnosis, appropriate investigations and a
management plan.

Patient (actor) briefing

 History of presenting complaint: intermittent headaches over the last 6 months. Headaches are
worse when bending and straining and in the morning.
 There has been a gradual deterioration in vision. When asked what part of vision is affected, she states
that it is peripheral vision loss. Her driving has deteriorated over the past 6 months also, almost
resulting in an accident.
 She has also put on weight over the same 6-month period despite no change in diet or activity.
 She has been aware that her face has become round and hairy. When seen by her GP, she was told
that her blood pressure was higher than it had ever been.(CUSHING)
 Past medical history: nil.
 Past drug history: no medication and no drug allergies.
 Social history: smokes occasionally and drinks approximately 10 units of alcohol a week.
 Family history: nil.
 Ideas, concerns and expectations: she is worried that she is going to die.
 Systems review: nil.

 The symptoms of increased ICP.

 Worse on bending over


 Often worse in the morning
 Deterioration in vision
 High blood pressure.

Q.1.Please summarise the history.


This is a 54-year-old shop keeper with a history of worsening headaches over 6 months. She has symptoms of
increased ICP including headache worse on bending over, deterioration in vision and has symptoms of
Cushing's syndrome, including weight gain and high blood pressure. She is an occasional smoker and drinks
socially.
Q.2.Please give a differential diagnosis and explain how you would justify the answer.
The features of the headache are of raised ICP with associated visual disturbances. The patient seems to be
describing peripheral field loss bilaterally (Bitemporal Hemianopia), therefore suggesting that the pathology
is in the region of the optic chiasm.
The differentials include:

 Pituitary adenoma
 Craniopharyngioma
 Meningioma.

Q. Please specify what management you would recommend.

 Perform an initial resuscitation (ABC approach) if the patient is compromised.


 Blood panel including pituitary profile.
 Give analgesia for the headaches.
 Initial plain CT to give indication of pathology and rule out other intracranial causes.
 MRI pituitary scan as guided by above investigations.
 Formal visual field tests performed by ophthalmology.
 Discuss with neurosurgical registrar/consultant.

Q. What hormones are released by the pituitary gland?


The anterior pituitary gland secretes:

 Growth hormone (GH)


 Thyroid stimulating hormone (TSH)
 Adrenocorticotropic hormone (ACTH)
 Prolactin
 Luteinising hormone (LH)
 Follicle-stimulating hormone (FSH).

The posterior pituitary secretes:

 Antidiuretic hormone (ADH)


 Oxytocin.

Q. What advice would you give the patient in terms of driving?


The patient should not drive and should disclose their condition to the DVLA. Once the pathology is confirmed
and treatment plan made, the duration of not being able to drive will be disclosed to her after correspondence
between the treating specialist and DVLA.
CASE 6

JAUNDICE

You are the on call SHO for general surgery. Mr P, a 65-year-old engineer, presents to Accident and Emergency
(A&E) as his wife has noticed that his skin has become yellow. You are to take an appropriate history in six
minutes (you may make notes). If you complete your history within the six minutes you should indicate to the
examiners that you are ready. In the remaining three minutes the examiners will ask you to present a
summary of the history. They may also ask you to discuss any particular physical signs you would look for on
examination, the likely differential diagnosis, appropriate investigations and a management plan.

Patient (actor) briefing

 His wife noticed that his skin was becoming yellow during the last week. He has also noticed that his
stools have been paler and more difficult to flush and his urine has been darker, over this period.
He has been feeling more tired and has had general malaise over the past three months. His appetite
has been reduced and he has lost one stone (6.35 kg) in weight. He has had some pain in the middle
of his abdomen which radiates to the back. The pain is gnawing and often wakes him at night.
Nothing makes the pain better, including the analgesia given by the GP.
 Past medical history: type 2 diabetes, which is controlled by diet.
 Medication history: analgesia for the pain as given by the GP, no known drug allergies.
 Social history: lives with wife and children, drinks 4 units of alcohol per week, smoked for 60 pack
years.

CONSIDER
The signs of obstructive jaundice.

 Pale stools
 Dark urine.

Some of the common symptoms of malignancy.

 Weight loss
 Malaise
 Tiredness (due to anaemia)
 Generalised pain
 Reduced appetite

Q.1.Please summarise the history.


Mr P is a 65-year-old engineer who presents with a one-week history of obstructive jaundice. The cause is
likely related to a malignancy based on a 3-month history of weight loss (one stone (6.35 kg), unintentional),
malaise and tiredness suggesting anaemia. A central abdominal pain suggests a mass-occupying lesion.
The pain radiates to the back and often wakes him at night. He has 60 pack year history of smoking.
Q.2.Please give a differential diagnosis and explain how you would justify the answer.
 This patient has obstructive jaundice (pale stools and dark urine due to no enterohepatic circulation).
 Differentials include:

 CBD stone
 Malignancy such as carcinoma of pancreatic head or cholangiocarcinoma.

Due to this patient’s weight loss, malaise and anorexia, I would want to exclude a malignant cause such as
Pancreatic malignancy.
Q.3.Please explain how the diagnosis would be confirmed.
 I would like to examine the patient’s abdomen to feel for any masses which may be present.
 Bloods: full blood count (FBC) to look for iron deficiency anaemia which can be related to
underlying malignancy, and raised white blood cells suggesting cholangitis.
 Urea and electrolytes (U&Es) to exclude Renal dysfunction due to jaundice.
 Liver function tests (LFTs) to look at the degree and type of jaundice: bilirubin (jaundice is defined
as elevation of serum bilirubin – but it is not clinically detectable until 40 µmol/l); alkaline
phosphatase (normal if pre-hepatic cause, mildly raised in hepatic causes, very high in post-
hepatic causes); aminotransferase (normal if pre-hepatic cause, very high in hepatic causes,
normal or mildly raised in post-hepatic causes).
 Amylase to check for acute inflammation of the pancreas.
 Clotting screen: prothrombin time is often prolonged.
 Tumour markers: (CA19-9)
 Imaging: Plain radiographs usually not helpful but may show: 10% of gallstones, which are radio-
opaque; calcification in chronic pancreatitis; air in the biliary tree (fistula between bowel and
biliary tree, previous ERCP, gas-forming organisms in severe cholangitis); soft-tissue mass in the
right upper quadrant (RUQ), which is an enlarged gallbladder.
 Ultrasound (essential baseline). It can show: gallstones in the gallbladder (but may miss duct
stones), dilated bile or pancreatic ducts, level and cause of obstruction, mass in the pancreas,
splenomegaly, ascites etc.
 MR cholangiopancreatography (MRCP): used to further investigate dilated ducts found on
ultrasound. It is replacing ERCP (which is now regarded as a therapeutic procedure) as the
investigation of choice as it has none of the risks of ERCP. It is also used to stage and assess
tumour operability.
 EUS (endoscopic ultrasound): ultrasound inserted into the duodenum endoscopically. Especially
useful in assessing the pancreatic head and bile duct and it is possible to perform a biopsy of a
periampullary mass. Also useful in picking up microlithiasis.
 CT scan to look for causes of extrinsic compression; formal staging CT can be performed if
malignancy suspected.
 Endoscopic retrograde cholangiopancreatography (ERCPQ.4.Please specify what management you
would recommend.

 Perform ABC and resuscitation if necessary.


 Full history and examination.
 FBC, U&E, CRP, LFTs, CA19-9, group and save.
 CT CAP with contrast.
 Urgent multidisciplinary team (MDT) follow up.

Q.5.To what level does a serum bilirubin need to rise before it becomes clinically detectable?
This needs to be approximately above 40 μmol/l before it becomes clinically detectable
Medications which can cause jaundice include:

 Hepatic jaundice: Paracetamol, Amiodarone, Diclofenac, fluconazole, Heparin, labetolol, diltiazem.


 Obstructive ‘cholestatic’ jaundice: Augmentin, flucloxacillin, erythromycin, septrin, captopril,
phenothiazides.
 A family history of blood disorders suggests a haemolytic disorder.

Examine:

 Perform essentially an abdominal examination. Inspect: in general for jaundice/cachexia, the chest for
gynaecomastia/spider naevi, and the abdomen for scars/masses.
 Hands: clubbing/palmar erythema/leukonychia
 Abdomen: palpate for organomegaly, other masses, ascites.
CASE 7.

ABDOMINAL PAIN HISTORY

Ms Edwards is a 32-year-old woman who you have been asked to see in A&E. She has presented with a one-
day history of abdominal pain and vomiting. You are to take an appropriate history in 6 minutes (you may
make notes). If you complete your history within the 6 minutes you should indicate to the examiners that you
are ready. In the remaining 3 minutes the examiners will ask you to present a summary of the history. They
may also ask you to discuss any particular physical signs you would look for on examination, the likely
differential diagnosis, appropriate investigations and a management plan.

Patient (actor) briefing

 Behaviour: in pain
 She is a 32-year-old woman with pain in the right upper quadrant (RUQ) but the pain radiates to the
epigastric region and to the back. The pain came on gradually. Originally, she had thought that it was
indigestion. The pain is intermittent and increases in ‘waves’. The waves are quite frequent but you
are unable to tell how often the pain arises.
 There is no vomiting but she is nauseous. There is no PR bleed. There is normal bowel habit.
 Past medical history: Gastro-oesophageal reflux ('heartburn') disease and hypercholesterolaemia.
 Past surgical history: appendectomy 8 years ago.
 Drug history: omeprazole, simvastatin, no known drug allergies.
 Social history: non-smoker, social drinker.
 Family history: father died age 87 from a myocardial infarction and mother is fit and well. No siblings.

Consider...
Suspected emergency presentations of gallstones – aside from pain, what other symptoms should be
explored?
Symptoms to be explored:

 Is the patient jaundiced?


 Have they been passing pale stool or dark urine?

History of presenting complaint (the patient may volunteer the following information but remember a
systematic approach to taking a pain history is SIROD CASP):

1. Site: ‘Where do you feel the pain?’ (localised or diffuse).


2. Intensity: ‘Can you rate the pain on a scale from 1 to 10. 1 being no pain and 10 the worst pain ever?’
3. Radiation: ‘Does the pain move anywhere else?’
4. Onset: ‘Did the pain come on suddenly or gradually?’
5. Duration: ‘How long does the pain last?’ (constant, colicky or related to posture).
6. Character: ‘What is the pain like?’ (dull, aching, sharp, knife-like).
7. Alleviating and exacerbating factors: ‘Does anything make the pain better, or worse? Does it relate to
what/when you eat?’
8. Symptoms associated with the pain: (see below).
9. Previous episodes: ‘Have you had a pain like this before?’ If yes: ‘What did you do then?’

You must ask about associated symptoms, these can be tailored somewhat to the presentation of pain:

 ‘When did you last open your bowels?’


 ‘Are you passing wind?’
 ‘Any pain passing urine?’ ‘Any frequency of urine?’
 ‘Any nausea or vomiting?’
 ‘When was your last menstrual period?’ ‘Any unusual discharge/bleeding from the vagina?’
 ‘Do you have a fever associated with the pain?’
 ‘Have you lost any weight recently?’
 ‘Any change of your bowel habit?'

Past medical and surgical history:

 ‘Any other medical problems?’


 ‘Any previous abdominal operations?’

Medication and allergies:

 ‘Do you take any regular medication?’


 ‘What medication, if any, do you take for your back pain?’
 ‘Do you have any allergies?’

Social history (smoking and drinking):

 ‘Are you a current or ex-smoker?’


 If yes, ask about smoking habits: ‘What do you smoke?’ ‘How many do you smoke a day?’ ‘How long
have you smoked for?’
 ‘Do you, or have you ever, drank alcohol?’
 If yes: ‘What type of alcohol do you drink?’ (eg spirits, wine, beer), ‘How much do you drink per week?’
(convert this to units per week). ‘How long have you been drinking for?’

Q. Please summarise the history.


This is a 32-year-old woman who presented with colicky right upper quadrant (RUQ) pain that has
progressively worsened and now radiates to the back and into the epigastrium. She has a past medical history
of reflux, takes omeprazole and a statin and has no known allergies. Her father died of a heart attack.
Q. Please give a differential diagnosis and explain how you would justify the answer.
Differential diagnosis:

 Gallstone disease: Pain and Patient demographic


 Peptic ulcer disease: on PPI non compliance, would fit a gastritis picture, ± ulcer, 'social
drinker'
 Pancreatitis: unlikely but pain radiating to the back, 'social drinker' is a complication of
gallstone disease
 Abdominal aortic aneurysm (AAA): must be considered in any abdominal pain radiating to the
back.

The differential for abdominal pain is very varied. Try to differentiate features of these in the history:

 Biliary tract disease: RUQ pain, may radiate to the shoulder, tender gallbladder, associated nausea and
vomiting, may be worsened or started following consumption of fatty food. Perform an ultrasound to
visualise the biliary tree and associated calculi – it is 90–95% sensitive for cholecystitis.
 Appendicitis: central pain migrating to the right iliac fossa (RIF), fever associated anorexia, nausea,
vomiting. Inflammatory markers are raised.
 Small bowel obstruction: general abdominal pain, distension, constipation and vomiting. Most
commonly caused by hernias or adhesions. Plain abdominal film shows distended loops of small
bowel; CT imaging confirms the diagnosis.
 Large bowel obstruction: general abdominal pain, distension, constipation and vomiting. Most
commonly caused by cancer, diverticular strictures or volvulus. Plain abdominal film shows distended
loops of large bowel.
 Pancreatitis: classically a boring central abdominal pain radiating to the back; however presentation is
varied. Associated with nausea, vomiting and dehydration. Serum amylase is three times above the
reference range. CT imaging is sometimes required to exclude the complications later during
admission.
 Peptic ulcer disease: a perforated peptic ulcer can present with severe abdominal pain and guarding.
Obtain an erect chest X-ray early to look for air under the diaphragm.
 Diverticulitis: left iliac fossa (LIF) pain with raised inflammatory markers.
 Mesenteric ischaemia: pain out of proportion to the physical examination, associated with vomiting
and diarrhoea. Ask about risk for embolic disease such as atrial fibrillation (AF) or valve lesions.
Arterial blood gases (ABGs) can be useful showing a metabolic acidosis and high lactate. Diagnosis is
often difficult and requires a high index of suspicion. Angiography is the gold standard investigation
 Ruptured abdominal aortic aneurysm (AAA): presents classically with severe abdominal pain
radiating to the back and profound hypotension; however, may present similarly to renal colic,
diverticulitis and acute myocardial infarction leading to a fatal delay in management. Urgent
resuscitation and surgery is required.

Non-abdominal causes:

 The classic central chest pain of a myocardial infarction may be felt lower down and ascribed to the
abdomen, particularly if an inferior infarct.
 Pulmonary embolism.
 Pneumonia.
 Pneumothorax.
 Diabetic ketoacidosis.
 Addisonian crisis.

Q. Please explain how the diagnosis would be confirmed.


Confirmation of diagnosis:

 Gallstones: ultrasound of liver/biliary tree


 Peptic ulcer disease: Oesophagogastroduodenoscopy (OGD) + urea blood test + erect chest X-ray
 Pancreatitis: amylase/lipase blood tests + CT abdomen
 Abdominal aortic aneurysm (AAA): may be identified by a bedside ultrasound scan but definitive
diagnosis is CT aorta (angiogram).

Q.Please specify what management you would recommend.

I would manage this patient with an ABC approach (and then it depends on candidate’s main differential; this
example is for cholecystitis):

 The patient should be kept nil by mouth and admitted.


 They should be optimised with oxygen if required, IV access (×2 large bore) and intravenous fluids
should be set up.
 If pyrexic, any signs of sepsis or raised inflammatory markers: IV antibiotics.
 A strict ‘input/output’ chart should be kept and the patient should be catheterised if circulatory
compromise is present.
 Appropriate analgesia should be prescribed and antiemetics should be written up.
 The patient should have thromboembolic stockings and the appropriate investigations should further
direct the management (ie as above).

Bedside investigations:

 Observations
 Urine dipstick (include pregnancy test)
 Blood tests: fbc, u&es, lfts, amylase, glucose, coagulation, group and save
 Ecg
 Arterial blood gases.
Other investigations:

 Erect chest X-ray


 Ultrasound scan
 Consider magnetic resonance cholangiopancreatography (MRCP) if LFT deranged/duct dilation
 CT scan if ultrasound scan is normal and pain has not resolved
 Oesophagogastroduodenoscopy (OGD) as outpatient appointment if no cause of pain can be identified.
CASE 8

P-R BLEEDING-1

You are the on-call SHO for general surgery, and A&E refer you Mr S, a 31-year-old electrician, who presents
with bleeding from his rectum. You are to take an appropriate history in 6 minutes (you may make notes). If
you complete your history within the 6 minutes you should indicate to the examiners that you are ready. In
the remaining 3 minutes the examiners will ask you to present a summary of the history. They may also ask
you to discuss any particular physical signs you would look for on examination, the likely differential
diagnosis, appropriate investigations and a management plan.

Patient (actor) briefing

 The blood is dark-red and is both clots and streaks. It has been occurring intermittently over the past
month. There is occasional associated lower abdominal pain. There is minimal blood on wiping, more
mixed in with the stools. No blood around the toilet bowel has been noted. Some mucus noticed. There
has been weight loss of one stone (6.35 kg), which is unintentional, and there has been no change in
diet.
 Stool frequency and consistency varies, sometimes loose. No-one else at home has suffered from the
same problems. There has been no recent travel. He feels run down and tired but otherwise fit and
well. There has been no vomiting, no swallowing problems, no shortness of breath, no headaches and
no chest pain.
 Behaviour: very concerned.
 Past medical history: nil.
 Past drug history: nil, no known drug allergies.
 History of presenting complaint: nil.
 Social history: he lives with his wife and does not smoke or drink alcohol.
 Ask about: diet/uncooked food.
 Family history: his grandfather had bowel problems from a young age but he cannot remember what
the condition was called.
 Systems review: nil

Consider...
Which features differentiate an upper GI bleed from a lower GI bleed.

 Upper GI bleed: dark, black tarry stool


 Lower GI bleed: fresh blood or clots mixed in with stool. If it is caused by haemorrhoids or anal
disease it may be covering the stool or on the paper.

What features might indicate a malignant cause.


Weight loss, malaise, tiredness (low Hb) and altered bowel habit might indicate a malignant cause.

Q.1.Please summarise the history.


This is a 31-year-old man who presented with per rectum (PR) bleeding for the past month; the blood is dark
and includes clots, mixed in with the stool and some on the paper. He has also noticed some mucus. He has
associated lower abdominal pain and has lost one stone (6.35 kg) in weight unintentionally. His bowel habit
has changed and is inconsistent. He is normally fit and well and lives with his wife
Q.2.Please give a differential diagnosis and explain how you would justify the answer.
Given that the patient has had a certain amount of ‘unintentional weight loss’ it is of paramount importance to
exclude a malignant cause for his symptoms. It is important to acknowledge that a malignancy is unlikely,
given the patient’s age and no positive, definite family history (however, it is essential to highlight that you are
aware of it to the examiner). Due to the patient’s age and change in stool habits with blood and mucus,
Inflammatory bowel disease (IBD) must be considered. Other differentials are:
 Polyps
 Anal fissure
 Haemorrhoids
 Diverticular disease.

Q.3.Please explain how the diagnosis would be confirmed.

 Abdominal examination including DRE.


 Routine bloods: FBC, U&E, CRP, LFTs, calcium, magnesium, phosphate, coagulation screen, group and
save (looking for raised inflammatory markers, dehydration, electrolyte disturbance secondary to
diarrhoea, albumin as a guide of nutritional status, coagulation defects).
 Stool sample: MCS, Faecal calprotectin.
 Colonoscopy is the gold standard, but flexible sigmoidoscopy can be performed acutely if the patient
has not been given bowel preparation.
 Abdominal radiograph for abdominal distention/symptoms of obstruction: assess for toxic megacolon,
obstruction, oedema.
 CT if concerning features found on examination (perforation/ obstruction) and for pre-operative
planning if surgery is indicated.
 MRI small bowel for possible Crohn's disease.

Q.4.What are the red flags associated with PR bleeding?

 Change in bowel habit


 PR bleed
 Tenesmus
 Anaemia: symptomatic or asymptomatic
 Weight loss
 Abdominal mass
 Bowel obstruction (usually cancers of the left side of the colon, as the faeces are more solid here and
thus more likely to obstruct).

Crohn's disease and ulcerative colitis

 Crohn’s disease: although most commonly found in the terminal ileum, Crohn’s disease can occur
anywhere in the alimentary canal, from the mouth to the anus. It may be confined to the large bowel,
or there may be involvement of both the small and large intestine. Perianal involvement is seen in
75% of patients.
 Ulcerative colitis: this is a mucosal disease that almost invariably involves the rectum and then
spreads more proximally in a continuous manner. Only 15% of cases extend more proximally than the
splenic flexure. (This group has a greater risk of complications, including cancer.) In a few cases, the
ileum is also affected (backwash ileitis).

Epidemiology:

 Both Crohn’s disease and ulcerative colitis are more common in developing countries and in younger
adults.
 Aetiology: unknown.
 There is family history in 20–30%.

Pathology:

 Crohn’s disease: can affect the GI tract from the mouth to the anus; in 70% of patients it involves the
small bowel. There is perianal involvement in 50–70%. There may be skip lesions of abnormal areas
with intervening normal mucosa. The whole thickness of the bowel is affected. There is a cobblestone
appearance of mucosa and fatty encroachment on serosa. Fistulation to adjacent organs is common.
There are non-caseating epithelioid granulomas in 60–70% of patients.
 Ulcerative colitis: occurs in the rectum and extends continuously proximally. It can affect the entire
colon with ‘backwash ileitis’ but no other area of the GI tract affected. Inflammation limited to
mucosa, not transmural. The serosa is not affected. There are inflammatory pseudopolyps and small
shallow ulcers. Granulomas are not typical.

Typical features of Crohn’s disease:

 Stricture formation leading to chronic intestinal obstruction


 Local perforation
 Abscess
 Fistula to exterior or other organs
 Colitis leading to diarrhoea, mucus and bleeding
 Anal fissures, ulcers, infections and skin tags
 Extraintestinal manifestations
 Anorexia, weight loss, malnutrition, anaemia, nausea.

Typical features of ulcerative colitis:

 Bloody diarrhoea with mucus, urgency and incontinence


 Constipation in cases of limited proctitis
 Cramping abdominal pain
 Anorexia, weight loss, malnutrition, anaemia, nausea
 Extraintestinal manifestations
 Important to recognise patients with severe acute colitis: severe local symptoms; frequency more than
10 stools/24 hours with blood; wasting, pallor, tachycardia, pyrexia; tender, distended abdomen;
these patients may progress to acute toxic dilatation of the colon and perforation.

Extraintestinal manifestations of inflammatory bowel disease related to disease activity:

 Skin: Pyoderma Gangrenosum, Erythema Nodosum


 Mucous Membranes: Aphthous Ulcers Of The Mouth And Vagina
 Eyes: Iritis
 Joints: Activity-Related Arthritis Of Large Joints.

Extraintestinal manifestations of inflammatory bowel disease unrelated to disease activity:

 Joints/Liver: Sacroiliitis; Ankylosing Spondylitis


 Biliary Tree: Chronic Active Hepatitis, Cirrhosis, Primary Sclerosing Cholangitis, Bile Duct Carcinoma
 Integument: Amyloidosis In Crohn’s Disease, Fingernail Clubbing.

Investigations:

 Sigmoidoscopy
 Colonoscopy
 Barium enema
 Small-bowel enema/barium meal
 MRI
 FBC
 ESR
 CRP
 Serum albumin
 Stool microscopy and culture.
Barium enema findings:

 Crohn’s Disease:

 There Is Discontinuous Distribution (‘Skip Lesions’),


 Rectal Sparing Is Common,
 ‘Cobblestone’ Appearance Of Mucosa, ‘
 Rose-Thorn’ Spiculation Ulcers,
 Fistulae
 Strictures.

 Ulcerative colitis: there is a

 Featureless ‘Hosepipe’ Colon,


 Decreased Haustrae
 It Affects The Rectum And Spreads Proximally,
 Mucosal Distortion
 Small Ulcers
 Pseudopolyps,
 Shortened Colon.

Medical management:

 This is a complex topic but try to remember the basic principle: that anti-inflammatories are
prescribed in a ladder of treatment for progressively severe or resistant inflammatory bowel disease.
 That ladder consists of 5-aminosalicylic acid preparations (for basic long-term control), steroids (for
short-term control of flare-ups), Azathioprine (for long-term control if becoming steroid-dependent)
and anti-tumour necrosis factor (TNF) biological agents such as infliximab (indicated in certain
complex cases).

Indications for surgery:

 To restore health in patients with chronic disease (eg in nutritional failure).


 To eliminate the risks of side-effects of steroids in patients requiring long-term high doses of steroids.
 Premalignant change seen on colonoscopic surveillance.
 Patients at high risk of developing cancer (ulcerative colitis with early onset, extensive colonic
involvement and continuous symptoms).
 To treat complications (usually emergency surgery) eg perforation, severe haemorrhage, toxic
dilatation (>6 cm megacolon), stricture causing obstruction, fistulation or abscess formation, sepsis. In
an acute severe attack, the diseased bowel needs to be defunctioned with an ileostomy.

Principles of surgery:

 In Crohn’s disease, surgery should be as limited as possible and be reserved for patients with a specific
operable problem, as it cannot be ‘cured’ by surgery, and post-operative complications are common.
 In ulcerative colitis (UC), radical surgery is often employed, because removal of the diseased segment
often cures the patient. Furthermore, patients are at risk of lethal toxic megacolon and have a tenfold
increased risk of developing carcinoma in long-standing disease. The most common operations
performed for UC are proctocolectomy with ileostomy, sphincter-preserving proctocolectomy with
ileal pouch, colectomy with ileorectal anastomosis, and subtotal colectomy with ileostomy and mucous
fistula.
 In PR bleeding cases, it is important to ask about pain, aggravating factors, stool consistency,
frequency and mucus. Also ask about associated vomiting, travel/dodgy food, ill contacts and what
treatments have been tried.
 If a patient already has inflammatory bowel disease (IBD), ask when the diagnosis was made, how
they were diagnosed, the last time investigations took place, the treatment so far, and consider extra-
abdominal features of IBD.

DIFFERENTIAL DIAGNOSES OF PR BLEEDING:

 MALIGNANCY
 INFLAMMATORY BOWEL DISEASE
 POLYPS
 ANAL FISSURE
 HAEMORRHOIDS
 INFECTIVE DIARRHOEA
 ANGIODYSPLASIA
 DIVERTICULAR DISEASE.
 INFECTIOUS GASTROENTERITIS (history will usually mention travel or eating new foods and the
change in bowel habit will be since then; can be mucus also).
CASE 9

PR BLEEDING-2

You are the surgical SHO in the colorectal clinic. You are about to see a 35-year-old man with per rectal (PR)
bleeding who has been referred to the clinic by the GP. You are to take an appropriate history in 6 minutes
(you may make notes). If you complete your history within the 6 minutes you should indicate to the examiners
that you are ready. In the remaining 3 minutes the examiners will ask you to present a summary of the history.
They may also ask you to discuss any particular physical signs you would look for on examination, the likely
differential diagnosis, appropriate investigations and a management plan

Patient (actor) briefing.

 He has been having per rectal (PR) bleeding for a month, and has passed clots and streaks, mixed in
with stools. There is no blood on wiping or in the toilet pan. Bowel habits have changed; they have
always been regular until the past few months. He has a feeling of urgency to pass stools at times. He
has lost a stone (6.35 kg) in weight, unintentionally, but his appetite has been less than normal. He is
feeling run down, tired and has less energy than usual.
 Behaviour: very concerned.
 Past medical history: nil.
 Past drug history: nil, no known drug allergies.
 Social history: he lives with his wife and children. He does not smoke and occasionally drinks alcohol.
 Family history: his grandfather had bowel cancer in his fifties.
 Systems review: nil other.

The red flags for PR bleeding.

 Change in bowel habit


 PR bleed
 Tenesmus
 Anaemia, symptomatic or asymptomatic
 Weight loss
 Abdominal mass
 Bowel obstruction (usually cancers of the left side of the colon, as the faeces are more solid here and
thus more likely to obstruct).

Q. Please summarise the history.


This is a 35-year-old man with PR bleeding and symptoms suggestive of a malignant cause. The blood appears
as clots and streaks mixed in with the stools. He has had a change in bowel habit, weight loss and symptoms of
anaemia. He has a positive family history for bowel cancer. He does not smoke and lives with his wife.
Q. Please give a differential diagnosis and explain how you would justify the answer.
It is of paramount importance to exclude a malignant cause for his symptoms. There is a family history of
bowel cancer and tenesmus with symptomatic anaemia, and weight loss and so malignancy is leading
differential. Other differentials to be excluded are:

 Haemorrhoids
 Anal Fissure
 Bowel Cancer
 Polyps
 Inflammatory Bowel Disease (Ibd)
 Infective Diarrhoea
 Angiodysplasia
 Diverticular Disease.
Q. Please explain how the diagnosis would be confirmed.
Investigations can be divided into bedside, laboratory, radiological and endoscopic.
Bedside investigations:

 Rigid sigmoidoscopy.
 Proctoscopy: this may show rectal tumours, and also help look for any other cause of any symptoms,
eg haemorrhoids, causing PR bleeding.

LABORATORY

 Full blood count (FBC) to look for anaemia, plus urea and electrolytes to assess renal function as the
patient may need a contrast CT scan, and a colonoscopy. Faecal occult blood testing is reserved for
screening.
 Tumour markers should only be considered for the surveillance period after treatment for cancer has
been given.

ENDOSCOPIC

Colonoscopy is the gold standard investigation for colorectal cancer as it provides good visualisation of
the tumour, allows for biopsies, assesses the colon for multiple lesions including polyps, and allows tattooing
of the tumour – this is done to help surgeons find the tumour during resection. Patients are required to take a
potent bowel preparation for this, and so those with a degree of renal failure, or who are generally quite frail,
should not undergo colonoscopy unless it is necessary.

IMAGING

 CT colonography can be considered for patients who are unsuitable or unable to tolerate colonoscopy.
CT enema and flexible sigmoidoscopy in combination is often used for elderly frail patients who
cannot have colonoscopy.
 CT is normally reserved for when a diagnosis of colorectal cancer is established, and it is usually
performed for staging purposes, ie CT of the chest, abdomen and pelvis.
 MRI of the pelvis is reserved for staging patients with confirmed rectal cancer. Endoanal ultrasound is
used in patients who cannot have MRI.

Q. Given the family history what tests may need to be considered?

 Genetic testing for siblings and children: if there is a family history of colorectal cancer at an early age,
ie less than 50 years, then immediate relatives of the affected should be considered for this.
 K-ras.

Q. Describe the blood supply to the colon.

 The blood supply to the colon comes from two arteries: the superior mesenteric artery (SMA) and the
inferior mesenteric artery (IMA). They are both branches of the aorta.
 The SMA divides into three branches: the ileocolic artery (supplies the terminal ileum, caecum,
appendix and proximal ascending colon); the right colic artey (supplies the ascending colon); and the
middle colic artery (essentially supplies the transverse colon).
 The IMA divides into the left colic artery (which joins the middle colic to supply the splenic flexure and
transverse colon) and the sigmoid artery (supplies the sigmoid and rectum).
 It is essential to understand the blood supply to the colon when resecting bowel for colorectal cancer.
This is because the lymph nodes need to be resected along with the affected part of the bowel. The
lymph nodes follow the same course as the arteries, thus these artery that supplies the affected part of
the bowel needs to be tied off, divided and resected too.
CASE: 11

PERIPHERAL VASCULAR DISEASE HISTORY

You are the surgical SHO in a District General Hospital, and this patient has been sent in by the GP as they are
not coping. Mr G, a 72-year-old retired plumber, has presented with bilateral lower limb pain. You are to take
an appropriate history in 6 minutes (you may make notes). If you complete your history within the 6 minutes
you should indicate to the examiners that you are ready. In the remaining 3 minutes the examiners will ask
you to present a summary of the history. They may also ask you to discuss any particular physical signs you
would look for on examination, the likely differential diagnosis, appropriate investigations and a management
plan.

Patient (actor) briefing

 He is a 72-year-old man who has been experiencing pain in both calves, but more so in the right calf.
The pain is an aching dull pain which worsens on exercise. It starts as a 5/10 but can be 8/10 at worst.
Initially, it was only on walking but lately he is also getting pain at rest. He also gets the pain at night
and hanging his right leg out of the bed helps to relieve the pain a little.
 He has not noticed any ulcerations on his legs. But he has noticed some discoloration.
 Past medical history: diabetes, hypertension and hypercholesterolaemia; stroke 2 years ago.
 Drug history: multiple medications and allergic to penicillin.
 Family history: lives with his daughter and she cares for him.
 Social history: 60 pack year smoking history and drink 30 units of alcohol per week.

To confirm the pain is claudication:

 ‘How far can you walk before you get the pain?’
 ‘Is it worse going uphill or in the cold weather?’ ‘
 Is it just in the calf or in the buttock and thigh as well?’ ‘
 How long does it take for the pain to go when you stop and rest?’ (eg seconds or several minutes?).

To assess severity and progression:

 How long have you been suffering from this pain on walking?’
 ‘Do you get pain that wakes you in the night?
 Do you swing your leg out of bed to help it go away?’
 ‘Do you get pain at rest?’
 ‘Have you ever had ulcers or sores on your leg?’
 ‘Do you get symptoms in your other leg?’

The complications diabetic patients can have.

 Peripheral neuropathy – leading to injury, ulceration and chronic infection


 Peripheral vascular disease
 Increased risk of infection.

The risk factors for peripheral vascular disease.

 Hypertension
 Unhealthy lifestyle (smoking, poor diet, obesity)
 Diabetes
 Hypercholesterolaemia
 Family history.
Q. Please summarise the history.
This is a 72-year-old man referred from his GP with increasing pain in his legs. He describes intermittent
claudication on exercise which is relieved with rest. He has pain at night which is relieved by increasing
perfusion. His symptoms are worsening and he is starting to get pain at rest. He has many risk factors for
peripheral vascular disease including smoking, high cholesterol, hypertension, diabetes and significantly he
had a stroke 2 years ago.
Q. The leading differential for this patient is
Bilateral peripheral vascular disease (PVD) with stenosis or occlusion in the superficial femoral artery (SFA),
with symptoms of critical limb ischaemia.
Q. Please explain how the diagnosis would be confirmed.

 I would examine limbs and ankle-brachial pressure index (ABPI).


 A duplex scan/CT angiography should be carried out to assess suitability for surgery/radiological
intervention.
 Contrast angiography prior to intervention or for patients unsuitable for iodine-based contrast.
 I would assess risk factors such as blood sugar, cholesterol, blood pressure and lipids.
 I would also assess co-morbidity such as renal, cardiac and lung function.

Q. The ABPI results are 0.3 in right leg and 0.5 in left leg, what do these values mean?
The clinical intervals for ABPI readings are:

 >0.9 = normal
 <0.8 = claudication/chronic ischaemia
 <0.4 = critical limb ischaemia.

Right leg = critical limb ischaemia.


Left leg = claudication/chronic ischaemia.
Q. What are the signs of acute limb ischaemia?

 Pain
 Paraesthesia
 Pistol shot onset
 Paralysis
 Pallor
 Pulseless
 Perishing cold

Q. Given the patient has critical ischaemia in the right leg what are the treatment options?

 Percutaneous Endovascular Revascularisation: balloon angioplasty and stenting. This is best in focal
SFA stenosis or Isolated iliac lesions.
 Surgery: Endarterectomy, Patch angioplasty, Bypass, eg femoral-popliteal bypass.

Examine:

 Look: general inspection for signs of cardiovascular disease and smoking; legs for pallor, venous
guttering, discoloration and ulceration.
 Palpation: Temperature, capillary refill, pulses.
 Auscultate for bruits.
 Special tests: Buerger’s test, ankle-brachial pressure index (ABPI).
 An ECG will be able to determine if the patient is an arrhythmia; patients with atrial fibrillation are at
risk of an embolic occlusion. An ECG is also necessary to help assess the patient’s risk from a cardiac
perspective, should they need a general anaesthetic.
CASE 11

CHEST PAIN HISTORY

This patient has been referred as an emergency to the surgical assessment unit by the patient’s GP. The
patient has developed upper abdominal and chest pain, which has been worsening over the last few hours,
with associated vomiting. You are to take an appropriate history in 6 minutes (you may make notes). If you
complete your history within the 6 minutes you should indicate to the examiners that you are ready. In the
remaining 3 minutes the examiners will ask you to present a summary of the history. They may also ask you to
discuss any particular physical signs you would look for on examination, the likely differential diagnosis,
appropriate investigations and a management plan.

Patient (actor) briefing


She is a 62-year-old woman with a 4-hour history of upper abdominal pain radiating to her chest. This has
been worsening, and she has vomited three times in the last 2 hours (no blood in vomitus). She has never had
this pain before, but does suffer from heartburn, which has been present for several months.
She has no previous history of heart disease, and is very fit and well. She is a non-smoker, rarely drinks
alcohol, and has no family history. She does suffer from arthritis in her knees and hips and has been taking
over-the-counter ibuprofen regularly over the last few months.
Q.Which two body systems you are going to enquire about and which of these is the most important.

 These symptoms may be relating to the cardiorespiratory system or may be caused by abdominal
pathology. Although often similar, a careful history will be able to tell these apart. Thus, careful
cardiorespiratory and abdominal history should be taken, rather than focusing on just one.
 The diagnosis to promptly assess for is a myocardial infarct, and even if it is clear that this is not the
diagnosis, you should ensure the examiner is confident that you have explored and excluded this as a
differential.

Consider...
Patients that may not volunteer information that they feel is unimportant, and assess for risk factors
associated with the pathology that they are considering.
This patient, for example, has been taking over-the-counter medication, which they may feel is not essential to
mention. However, NSAIDs are a chief risk factor for peptic disease, and this must be extracted from the
history.

Q. Please summarise the history.


This is a 62-year-old woman with a 4-hour history of worsening upper abdominal and chest pain. It is the first
occurrence of such symptoms, but she has suffered from dyspepsia over the last few months. There is no
radiation of pain to arms or jaw, no associated respiratory symptoms, and no previous cardiac history. She has
associated vomiting, but no haematemesis. She is otherwise fit and well, but takes ibuprofen regularly for mild
arthritis.
Q. Please give a differential diagnosis and explain how you would justify the answer.

 The diagnosis to exclude most importantly is an acute myocardial infarct.


 However, the patient’s history is suggestive of a peptic ulcer, which is likely to have perforated.
 This is based on no previous cardiac history or risk factors, and the patient has been taking ibuprofen
with a long history of dyspepsia and heartburn.
Q. Please explain how the diagnosis would be confirmed.

 Assess the baseline observations.


 Perform a careful physical examination, particularly to assess the cardiorespiratory systems and the
abdomen.
 Perform an ECG to assess for any acute changes.
 Take samples for laboratory tests to assess for anaemia, inflammatory markers and troponin (a
negative test would be reassuring and likely exclude a myocardial infarct).
 Perform an erect chest X-ray to assess for a perforated viscus – if negative, one should consider a CT
scan.

Q. Please specify what management you would recommend.

 First, a myocardial infarct should be excluded by a negative examination and troponin, and a normal
ECG.
 If there is tenderness/peritonism on examination, then an erect chest X-ray may reveal a
pneumoperitoneum. If negative (and mostly even if positive) a CT scan should be performed to assess
for the site of perforation (in this case likely duodenal or gastric).
 IV antibiotics, proton pump inhibitor, analgesia and anti-emetic should be administered.
 If the patient is unwell, a urinary catheter should be inserted.
 The patient should be kept NBM and IV fluids administered until a conclusive diagnosis reached, and
the further management plan (conservative or operative treatment) is established.
CASE 12

BREAST LUMP HISTORY-1

You are surgical trainee covering the breast clinic today. You have been asked to take a history from a 60-
year-old woman with a new lump in her right breast. You are to take an appropriate history in 6 minutes (you
may make notes). If you complete your history within the 6 minutes you should indicate to the examiners that
you are ready. In the remaining 3 minutes the examiners will ask you to present a summary of the history.
They may also ask you to discuss any particular physical signs you would look for on examination, the likely
differential diagnosis, appropriate investigations and a management plan.

Patient (actor) briefing

 Behaviour: concerned
 She found the lump in her right breast 2 weeks ago while showering. She thinks it may have gotten a
little bigger since she first found it. She doesn’t check her breasts regularly and doesn’t know how long
it has been there. It is non-tender. She has not noticed any other lumps in her breasts or armpits. No
skin changes have been noticed and no nipple discharge.
 Periods started aged 11 years and finished at 60 years old.
 She is currently on hormone replacement therapy (HRT). She has no children and lives with her
husband.
 She is very anxious as her mother had breast cancer aged 70 years old and so she has not attended any
screening.
 She is otherwise fit and healthy up until this point, with no other medications and no allergies.
 She does not smoke or drink alcohol.

Q. Please summarise the history.


This is a 60-year-old woman with new-onset lump in her right breast, first noticed 2 weeks ago and it has
enlarged since then. She has not noticed any discharge or skin changes. She has a family history of breast
cancer. She is post menopausal and otherwise has no other medical problems and is not on any long-term
medications
Q. How would you manage this patient?
I would be concerned to exclude breast cancer in this lady, who has several risk factors. I would want to
examine her as part of triple assessment to include:

 EXAMINATION
 IMAGING (ULTRASOUND SCAN OR MAMMOGRAM)
 FINE NEEDLE ASPIRATIONS (FNAC).

Q. List what you would be looking for on examination of the breast?


Character of the lump:

 Location
 Size
 Duration
 Method of discovery
 Hormonal/cyclical influence
 Pain
 Nipple discharge.

Q. What are the risk factors for breast cancer?


The risk factors for breast cancer:

 Early menarche <12 years


 Late menopause >55 years
 Nulliparous
 First pregnancy >30 years
 Increased oestrogen levels
 Never breastfed
 Currently using oral contraceptive (ocp)/hormone replacement therapy (hrt)
 >5 years hrt use
 Family history of breast cancer
 Previous breast cancer, ductal carcinoma in situ (dcis) or local carcinoma in situ (lcis)
 Smoking.

Q. The triple assessment reveals carcinoma of the breast, what are the management options?
Breast cancer can be managed through surgery, radiotherapy and adjuvant treatment such as: chemotherapy,
oophorectomy, other medications (eg tamoxifen and aromatase inhibitors) and immunotherapy. Management
depends on the tumour type.
Surgery for breast cancer:

 Excision biopsy: to diagnose, not to treat, eg in lesions suspicious on mammography but not palpable
or borderline suspicious FNA, but mammogram and clinical examination suggest benign disease. May
need wire localisation under mammographic control pre-operatively if impalpable screen-detected
lesion. Should remove no more than 20 g of tissue (not aiming for complete clearance as it is for
diagnosis, not treatment). Will need definitive treatment if histology is positive and clearance is not
complete.
 Wide local excision (breast conservation surgery): single, small (<4 cm diameter in small breast)
node – negative local disease. Aim is cure with good clearance. If margins not clear, perform
mastectomy.
 Mastectomy: large/central/multifocal disease or patient’s preference. Also may be indicated in
recurrence or inadequate clearance (affected margins) after wide local excision.
 Axillary node sampling: used if the axilla is disease-free on pre-operative imaging. At least four
palpable nodes are sampled from the axilla. If these contain metastatic disease the patient will require
axillary clearance or radiotherapy to treat axilla.
 Sentinel node biopsy: another technique used if the axilla is disease-free on pre-operative imaging. It
aims to remove the sentinel nodes (first lymph nodes to drain the breast). A radiolabelled colloid ±
blue dye is injected into the breast – a probe is used to find ‘hot’ and ‘blue’ nodes. If no histologically
involved nodes, no further treatment needed. If involved node(s) are found, need axillary clearance or
radiotherapy to treat axilla.
 Axillary clearance: removing level 1, 2 and 3 nodes (lateral, behind and medial to pectoralis minor).
 Even if nodes are positive, radiotherapy not needed after level 3 clearance unless recurrence detected
later.
 Complications of mastectomy: seroma, infection, flap necrosis.
 Complications of axillary surgery: nerve damage (loss of sensation inner aspect upper arm due to
intercostobrachial nerve injury), lymphoedema, wound infection and reduced range of shoulder
movement.

Radiotherapy:

 Breast: all patients after breast conservation surgery. High-risk mastectomy patients (pectoralis major
involvement or any two of the following: axillary lymph node involvement, lymphatic or vascular
invasion, grade 3 cancer, tumour >4 cm diameter).
 Axilla: if sentinel lymph node biopsy/axillary sampling shows positive nodes or in axillary recurrence
if no previous radiotherapy (can only be given once).
 Complications of radiotherapy: skin reactions, telangiectasia, cardiac damage, pneumonitis,
osteoradionecrosis and lymphoedema in axillary radiotherapy (especially when combined with
axillary surgery).
Adjuvant treatment:

 Chemotherapy: most effective in premenopausal patients. Considered if lymph node positive,


oestrogen receptor (ER) negative, grade 3, large tumour, lymphovascular invasion. Cyclophosphamide,
methotrexate and fluorouracil are common agents used in the UK. Side-effects include fatigue, alopecia
and nausea.
 Oophorectomy (or ovarian ablation by radiotherapy or gonadotropin-releasing hormone): only of
benefit in premenopausal women.
 Tamoxifen (selective oestrogen receptor modulator): first-line treatment for pre/post menopausal
ER+/progesterone receptor (PR)+ patients with invasive breast cancer. History of thrombosis is
relative contraindication. Side-effects include vaginal dryness, loss of libido and hot flushes.
 Aromatase inhibitors: block the peripheral conversion of androgens to oestrogen. Only used in
postmenopausal women, eg anastrozole, exemestane and letrozole.
 Patients who are elderly and unsuitable for general anaesthetic with ER+ breast cancer can be treated
by primary endocrine therapy to control disease progression.

CASE 13

BREAST LUMP-2

You are surgical trainee covering breast clinic today. You have been asked to take a history from Miss W, a 55-
year-old student teacher. She noticed two lumps in her left breast, two weeks ago. You are to take an
appropriate history in six minutes (you may make notes). If you complete your history within the six minutes
you should indicate to the examiners that you are ready. In the remaining three minutes the examiners will
ask you to present a summary of the history. They may also ask you to discuss any particular physical signs
you would look for on examination, the likely differential diagnosis, appropriate investigations and a
management plan.

Patient (actor) briefing

 She found the two lumps in her left breast two weeks ago. They are both 2 × 2 cm in size, and they are
very mobile. She does not check her breasts regularly and so she does not know how long they have
been there. They are both non-tender. She has not noticed any other lumps or bumps in her breasts or
armpits.
 There are no obvious skin changes and no nipple discharge.
 She started her periods at age 15 and they are regular.
 She is currently on the oral contraceptive pill (OCP). She has no children and lives with her friends.
 She does not take any other medications and has no allergies.
 There is no family history of breast cancer.
 She does not smoke or drink.

Some signs that may make you think of a malignancy.


Signs include:

 Fast growing
 Irregular shape
 Skin changes
 Immobile/tethered to skin.

Q. Please summarise the history.


This is a 55-year-old woman, with a 2-week history of two discrete mobile lumps in the left breast. They are
not painful and have no associated nipple discharges or skin changes.
Q.2Please give a differential diagnosis and explain how you would justify the answer.
I suspect this woman has two fibroadenomas due to their mobility ('mobile mice'); however for all breast
lumps we must consider the following differentials.
Generalised swelling:

 Pregnancy
 Lactation
 Puberty
 Mastitis.

Discrete breast lump:

 Fibroadenoma
 Fibrocystic disease
 Simple cyst
 Abscess
 Fat necrosis
 Galactocele
 Duct papilloma.

Discrete malignant breast lump:

 Carcinoma of the breast


 Phylloides tumour
 Sarcoma/lymphoma.

Q.3.Please specify what management you would recommend.


For all patients presenting with a breast lump, I would like to examine her as part of the triple
assessment. Triple assessment:

 clinical examination
 imaging
 fine needle aspiration (FNA).

The imaging depends on age, if the patient is less than 35 years old, an ultrasound can is carried out as breast
tissue is too dense for mammograms.
All patients over 35 years are imaged using mammograms.
Fibroadenomas do not need to be excised if definitively diagnosed on triple assessment but the patient may be
happier with the lump excised.
Q.4.What is a fibroadenoma?

 It is the most common benign tumour of the breast.


 It is new growth of fibrous and glandular tissue.
 It is common in premenopausal women.
 It may change with menstrual cycle.
 A well-circumscribed lump.
 It clinically feels like a mobile, smooth, 'breast mouse'.
 Histologically, there is characteristic delicate cellular fibroblastic stroma enclosing glandular
cystic spaces within an epithelial lining.
 There is typical mammographic and ultrasonographical appearance.
CASE.14

NECK LUMP HISTORY

You are the SHO on a busy ENT rota. You have been called to clinic to assist Mrs Jones, a 46-year-old woman
who has presented with a neck lump. You are to take an appropriate history in six minutes (you may make
notes). If you complete your history within the six minutes you should indicate to the examiners that you are
ready. In the remaining three minutes the examiners will ask you to present a summary of the history. They
may also ask you to discuss any particular physical signs you would look for on examination, the likely
differential diagnosis, appropriate investigations and a management plan.

Patient (actor) briefing

 The lump has been growing over the last month and she has noticed that her voice has changed. The
lump sits in the midline of her neck, is not painful and does not give any difficulty swallowing.
 Her voice has become very slightly deeper and slightly hoarse. It was her family that commented on
her voice change.
 She has also lost some weight.
 Associated symptoms: no change in vision or eyes, some hot flushes, no changes in menstruation
pattern and no changes in bowel habits.
 Past medical history: asthma.
 Drug history: salbutamol inhaler/no known allergies.
 Family history: nil.
 Social history: lives with family, has smoked 20 cigarettes per day for the past 10 years. She does not
drink alcohol. She works at a school

Q: What changes would be associated with increased hormone excretion.

Changes associated with increased hormone excretion are hot flushes, menstrual changes (heavy bleeds,
increased pain at menstruation), Hirsutism and Vocal changes.
Q.1. Please summarise the history.
Mrs Jones is a 46-year-old woman who has a midline neck lump that is increasing in size, her voice had
become deeper and hoarse but it has not affected her swallowing and is not painful. Since the lump appeared
she has lost one stone (6.35 kg) in weight. She is a smoker with a 10 pack year history.
Q.2.Please give a differential diagnosis and explain how you would justify the answer.

 Thyroid carcinoma
 Inflammatory goitre: Hashimoto’s thyroiditis and DeQuervain’s thyroiditis
 Toxic goitre: Graves’ disease, solitary toxic adenoma, toxic multinodular goitre, hypothyroidism.

Q.3.Please explain how the diagnosis would be confirmed.

 History
 Full examination
 Ultrasound imaging
 Fine needle aspiration or biopsy.

Q.4The triple assessment of Mrs Jones is highly suggestive of a medullary cell carcinoma. She undergoes a
total thyroidectomy under your consultant’s care. What are the complications of a thyroidectomy?

Immediate complications:

 Haemorrhage
 Hoarseness due to recurrent laryngeal nerve (RLN) injury or, more commonly, laryngeal oedema
 Stridor and respiratory distress due to bilateral RLN injury or paratracheal haematoma
 Myxodema crisis.

Early complications:

 Infection
 Seroma
 Hypoparathyroidism leading to hypocalcaemia
 Hypothyroidism.

Late complications:

 Recurrence of malignancy
 Keloid or hypertrophic scarring.

Q.History of presenting complaint (the patient may volunteer the following


information but questions you need to cover include):

 ‘Do you prefer a warm or cold room?’


 ‘Have you gained or lost any weight recently?’ If yes: ‘How much and over how long?’
 ‘Has your appetite increased recently?’
 ‘Has your bowel habit changed?’
 ‘Have you noticed a change in your mood?’
 ‘Have you noticed any palpitations or chest pain?’
 ‘Have you noticed a change in your periods?’ (to female patients only).
 ‘Have you noticed a change in your appearance?’
 ‘Have you had a change in your vision?’
 ‘Have you noticed a lump in your neck?’ If yes: ‘Does it cause you any problems?’ Local complications
of a goitre may include stridor, superior vena cava (SVC) obstruction, dysphagia, sudden painful
enlargement (haemorrhage into cyst).
CASE.15

GROIN LUMP HISTORY

This patient has been referred by the emergency medicine registrar with a suddenly painful, large right groin
lump. He has attempted to reduce it, but it is too painful for the patient. You are to take an appropriate history
in 6 minutes (you may make notes). If you complete your history within the 6 minutes you should indicate to
the examiners that you are ready. In the remaining 3 minutes the examiners will ask you to present a
summary of the history. They may also ask you to discuss any particular physical signs you would look for on
examination, the likely differential diagnosis, appropriate investigations and a management plan.

Patient (actor) briefing


He is a 50-year-old man who has noticed a spontaneously reducing lump in the right groin over last 6 months.
It has not caused any trouble. Today, the lump emerged and did not reduce, and has become increasingly
painful. His bowels are opening, although he has been constipated recently. There has been no vomiting or
high temperature. His background includes:

 smoker 10/day for 30 years with occasional cough


 no past medical or surgical history
 no family history
 active and enjoys going to the gym on occasion.

Q. What other concerning causes of a groin lump you should assess for.
Any groin pain should include an assessment of the genito-urinary system to assess for a testicular torsion.
Apart from this, a femoral aneurysm, saphena varix or abscess should be assessed for.
-If this is indeed a hernia, what important points in the history it is important to assess for.
You want to know, as it will affect management, does this acute hernia contain ischaemic or obstructed bowel?
Thus, features such as generalised abdominal pain, vomiting and fever are important to enquire about. Also,
the underlying cause and impact on post-repair recovery should be explored, in particular, a history of
physical activity, cough, smoking, and constipation – all of which may raise intra-abdominal pressure.
Q. Please summarise the history.
This is a 50-year-old physically active man with a 6-month history of noticing a spontaneously reducible mass
in his right groin. It has been acutely painful and irreducible over the last 3–4 hours. There is no associated
generalised abdominal pain, vomiting or urinary symptoms. He is a smoker with a chronic cough and a
tendency to constipation, although passing flatus, and has had a bowel movement within last 24 hours. There
have been no previous similar symptoms and no past medical history.
Q. Please give a differential diagnosis and explain how you would justify the answer.
Important potential diagnoses to exclude are:

 Femoral aneurysm: this has a longer history of a slowly enlarging mass, not spontaneously reducible,
and associated lower limb insufficiency.
 Saphena varix: this spontaneously reduces (positional), there is a longer history, it is unusually non-
tender, and is associated with lower limb venous disease.
 Abscess: there is a short history of a rapidly enlarging mass, and skin changes and fever are present.
 Testicular pathology: the pain is mostly in testes/scrotum for torsion and orchitis, urinary symptoms
are often present, and there is swelling in the scrotum rather than groin.

Although it may be difficult to differentiate between an inguinal and femoral hernia from the history, an
inguinal hernia is more common and is more likely to be present for a long time, as in this case, compared to a
femoral hernia.
Q. Please explain how the diagnosis would be confirmed.
Diagnosis confirmation:
 Check observations.
 Perform a physical examination of the groin, abdomen, and external genitalia, as well as an assessment
of the lower limbs.
 Obtain laboratory tests to assess for inflammatory markers and renal function. The lactate should be
measured too.
 Perform an abdominal X-ray to assess for any evidence of bowel obstruction.
 If any diagnostic doubt, an ultrasound or CT of the groin can be performed to confirm a hernia and
exclude vascular causes.

Q. Please specify what management you would recommend.

 After examination, analgesia should be administered.


 If this is indeed an incarcerated hernia, this patient should undergo emergency repair. The patient
should be booked for emergency surgery, and consented for the operation.
 Of note, the need for a laparotomy and bowel resection should be included in the consent.
 The patient should be informed of the need to avoid strenuous activity in the first 4–6 weeks post-
surgery to reduce the risk of a repair dehiscence.

The lump itself:

 ‘How long have you had a lump in the groin?’


 ‘Does it move in and out of its own accord?’ ‘Have you ever had to push it back in yourself?’
 ‘Has it ever become stuck outside before, where you required medical assistance?’
 ‘Has it been increasing in size or causing you any pain?’
 ‘Do you have any pain or changes in skin in the leg?’
 ‘Has the presence of the lump impeded your ability to walk or exercise?’
 ‘Has there been any trauma or injury to the groin or leg?’
 ‘Is there any pain in the testicle/scrotum or any urinary symptoms?’

Important to specifically assess for, once it is clear this is a hernia:

 ‘How long has it been stuck outside for?’


 ‘Is there any radiation of the pain into your abdomen?’
 ‘If so, when did this start, and how severe is this?’
 ‘Has there been any vomiting?’
 ‘When did you last open your bowel or pass flatus?’
 ‘Have you suffered any high temperatures with the pain?’
 ‘Have you had any similar symptoms on the other side?’
 ‘When did you last eat or drink?’ (planning for possible emergency surgery).

CASE 17

LIMPING CHILD HISTORY

You are an ST in orthopaedics and have been asked to see a 5-year-old boy with his mother. You are to take
an appropriate history in 6 minutes (you may make notes). If you complete your history within the 6 minutes
you should indicate to the examiners that you are ready. In the remaining 3 minutes the examiners will ask
you to present a summary of the history. They may also ask you to discuss any particular physical signs you
would look for on examination, the likely differential diagnosis, appropriate investigations and a management
plan.

Patient (actor) briefing


He has no previous history other than a recent sore throat. For the past 4 days, he has experienced right hip
and knee pain and has been limping on his left side. There is no history of trauma. He had a normal vaginal
delivery with no complications. He is usually fit and well. He has one sister who also had a cold and sore throat
recently. His father was taken to hospital for a ‘sore leg’ when he was young but he can’t remember why.
Consider...
What should always be in the back of your mind when seeing children with musculoskeletal problems,
particularly if the history from the child and parent does not match the presentation.
Non-accidental injury should be considered.
Q.1 What is the most common cause of hip pain in children and what symptoms may this child have had other
than hip pain?
The most common cause is transient synovitis of the hip. It may be associated with low grade-fever, thigh or
knee pain and signs of infection elsewhere (eg urinary tract infection).
Q.2 What initial investigations may you want to perform on this patient following your history and
examination?

 Bloods to include full blood count (FBC), C-reactive protein (CRP) and erythrocyte sedimentation rate
(ESR)
 X-rays of the hip and possibly the knee
 Ultrasound.

Q.3.What is Hilton’s law?


Hilton’s Law: the nerves that innervate a joint usually innervate the muscles that act on it and the skin over
their attachments. Pain from the hip joint may be referred to any area that is supplied by a nerve innervating
the muscles that cross the joint.
Q.4.Radiological examination shows a subchondral crescent-shaped radiolucent line. What is the most likely
diagnosis?
The most likely diagnosis is Perthes disease. This is a comparatively rare paediatric condition where the blood
supply to the head of the femur is disrupted thereby causing avascular necrosis
Q.5What diagnosis may you want to consider in a 13-year-old obese boy with history of acute-on-chronic pain
in the hip?
Slipped upper femoral epiphysis (SUFE) should be considered. This condition tends to occur in adolescents
that are still growing where the head of the femur slips off of the femoral neck.
CASE 18
KNEE PAIN HISTORY- MRCS CHENNAI-2018 SEPTEMBER
Mr W, a 78-year-old retired accountant, presents with worsening knee pain. You are to take an appropriate
history in 6 minutes (you may make notes). If you complete your history within the 6 minutes you should
indicate to the examiners that you are ready. In the remaining 3 minutes the examiners will ask you to present
a summary of the history. They may also ask you to discuss any particular physical signs you would look for
on examination, the likely differential diagnosis, appropriate investigations and a management plan.

Patient (actor) briefing

 He is a 78-year-old retired accountant with worsening right knee pain. It is worse after use, especially
after climbing stairs. No pain at night. Knee stiffens up as you rest it. It occasionally swells but has
never given way or locked. Had a knee injury playing football 20 years ago but cannot remember the
details. He has bilateral hip pain but, this is not as severe as the knee pain.
 Past medical history: Asthma and TURP operation on prostate 2 years ago (no problems with the
anaesthetic).
 Drug history: Salbutamol inhaler as required/cannot take NSAIDs as give heartburn.
 Social history: lives with wife, non-smoker and no alcohol. Lives in a fourth-floor flat.

Q. Please summarise the history.


This is a 78-year-old accountant, with knee pain and swelling after exercise. It does not lock or give way. The
hips are also painful but not as much as the knee. He has a pertinent history of prostate cancer and asthma,
although he is not on steroids for his asthma.
Q. Please give a differential diagnosis and explain how you would justify the answer.
It is most likely that this man has osteoarthritis (OA) of the right knee, secondary to the previous injury (most
likely meniscal). As the patient mentions other joints causing pain, other forms of arthritis such as rheumatoid
and psoriatic arthritis need to be ruled out.
Q. Please explain how the diagnosis would be confirmed.

 I would like to examine the patient and X-ray the knee, with the patient standing and fully weight
bearing in the knee.
 As with all joints, two views are necessary: anteroposterior (AP) and lateral.

RADIOGRAPHIC SIGNS OF OSTEOARTHRITIS INCLUDE:

 LOSS OF JOINT SPACE


 OSTEOPHYTE FORMATION
 JUXTA-ARTICULAR SCLEROSIS
 SUBARTICULAR BONE CYSTS.

Q. Please specify what management you would recommend.

 Conservative management: quadriceps exercises, analgesia and walking aids.


 Joint injections: steroid injections.
 Surgery: realignment osteotomy of the tibia (in younger patients with only one compartment
affected).
 Arthroplasty: can be unicompartmental, patella-femoral replacement or whole tibiofemoral
articulation replacement. This is considered when conservative management has failed and the pain,
stiffness and immobility affect the patient’s quality of life.

Potrebbero piacerti anche